Find equation of circle in the first quadrant touches $x$-axis $y$-axis and straight line $3x-4y-20=0$ . The...











up vote
0
down vote

favorite












1)Find equation of circle

2)Equation of another tangent from point $H$ to the circle



The circle in the first quadrant touches $x$-axis $y$-axis and straight line $3x-4y-20=0$. The point $H(12,4)$ lies on the straight line.
How should I proceed to get the equation?



$y=(3/4)x-5$. I know $c=-5$. Correct me if I am wrong?










share|cite|improve this question
























  • Right up to there.
    – Aretino
    Jan 11 '16 at 17:06










  • I do not understand.how can i find the center and radius
    – Ameet
    Jan 11 '16 at 17:10















up vote
0
down vote

favorite












1)Find equation of circle

2)Equation of another tangent from point $H$ to the circle



The circle in the first quadrant touches $x$-axis $y$-axis and straight line $3x-4y-20=0$. The point $H(12,4)$ lies on the straight line.
How should I proceed to get the equation?



$y=(3/4)x-5$. I know $c=-5$. Correct me if I am wrong?










share|cite|improve this question
























  • Right up to there.
    – Aretino
    Jan 11 '16 at 17:06










  • I do not understand.how can i find the center and radius
    – Ameet
    Jan 11 '16 at 17:10













up vote
0
down vote

favorite









up vote
0
down vote

favorite











1)Find equation of circle

2)Equation of another tangent from point $H$ to the circle



The circle in the first quadrant touches $x$-axis $y$-axis and straight line $3x-4y-20=0$. The point $H(12,4)$ lies on the straight line.
How should I proceed to get the equation?



$y=(3/4)x-5$. I know $c=-5$. Correct me if I am wrong?










share|cite|improve this question















1)Find equation of circle

2)Equation of another tangent from point $H$ to the circle



The circle in the first quadrant touches $x$-axis $y$-axis and straight line $3x-4y-20=0$. The point $H(12,4)$ lies on the straight line.
How should I proceed to get the equation?



$y=(3/4)x-5$. I know $c=-5$. Correct me if I am wrong?







circle






share|cite|improve this question















share|cite|improve this question













share|cite|improve this question




share|cite|improve this question








edited Feb 28 at 14:24









J. M. is not a mathematician

60.4k5146283




60.4k5146283










asked Jan 11 '16 at 16:31









Ameet

286




286












  • Right up to there.
    – Aretino
    Jan 11 '16 at 17:06










  • I do not understand.how can i find the center and radius
    – Ameet
    Jan 11 '16 at 17:10


















  • Right up to there.
    – Aretino
    Jan 11 '16 at 17:06










  • I do not understand.how can i find the center and radius
    – Ameet
    Jan 11 '16 at 17:10
















Right up to there.
– Aretino
Jan 11 '16 at 17:06




Right up to there.
– Aretino
Jan 11 '16 at 17:06












I do not understand.how can i find the center and radius
– Ameet
Jan 11 '16 at 17:10




I do not understand.how can i find the center and radius
– Ameet
Jan 11 '16 at 17:10










3 Answers
3






active

oldest

votes

















up vote
0
down vote













1) We want to find the $x-$ and $y-$intercepts of the line so that we can use geometry to solve for the radius and center. These are simply $(0, -5)$ and $(frac{20}{3}, 0).$ The hypotenuse of the right triangle formed is $frac{25}{3}$ by Pythagoras. The circle divides each side of the triangle into two parts, and every pair of sides shares a segment. Call the radius $r$ and the other segments $a$ and $b.$ Then we have $r + a = frac{20}{3},$ $r + b = 5,$ and $a + b = frac{25}{3}.$ Solving, we have $a + b + r = 10,$ so $r = 10 - frac{25}{3} = frac{5}{3}.$ This means the center is at $(frac{5}{3},-frac{5}{3}).$ This is actually in Quadrant IV - it is impossible for the circle to be in Quadrant I. The equation of the circle is thus $boxed{(x - frac{5}{3})^{2} + (y + frac{5}{3})^{2} = frac{25}{9}}.$






share|cite|improve this answer






























    up vote
    0
    down vote













    Since the circle touches x-axis and y-axis its center must lie on line $x=y$. Its coordinates must be $ (h,h)$. Equation of circle is



    $$ (x-h)^2 +(y-h)^2 = h^2 $$



    It should satisfy line L $$ y = 3 x/4 -5 $$



    Eliminate y:



    $$ x= 4/25 (15+7 h+2 sqrt 2 sqrt{-50-5 h+3 h^2}) $$



    The quantity under radical sign should vanish for a coincident root at tangent point, or ,



    $$ -50-5 h+3 h^2 = 0 $$



    which has roots:



    $$ h = 5 , h = -10/3 $$



    The latter is discarded as the circle must be in first quadrant.



    So the equation of the desired circle is finally:



    $$ (x-5)^2 +(y-5)^2 = 5^2 = 25 $$



    Point of tangential contact $(8,1)$ (not asked for).






    share|cite|improve this answer




























      up vote
      0
      down vote













      For question 1) we look for circles of the form $(x-a)^2+(y-a)^2=a^2$, since these are tangent to the axes, and the only ones because a tangency to the axis determines the radius line.



      ($(x-a)^2+(y+a)^2=a^2$ gives the circles in the 2. and 4. quadrants)



      The problem is to get the third tangency with the given line. First the point needs to be on the circle: $(x-a)^2+((3/4)x-5-a)^2=a^2$ or $(frac{25}{16}x^2+(-frac{7}{2}a-frac{15}{2})x+a^2+10a+25=0$, and it should touch doubly, i.e. the discriminant should be 0. This gives us a 2. order equation in a. In fact it is $(-frac{7}{2}a-frac{15}{2})^2-4(frac{25}{16})(a^2+10a+25)=2 (a - 5) (3 a + 10)=0$, so that $a=5$ or $a=-10/3$ a solution in the third quadrant.



      Circle bundle varying with a



      So the solution is $(x-5)^2+(y-5)^2=5^2$ which goes through $(8,1)$ on the line which is a $3,4,5$-triangle from the center $(5,5)$.



      For question 2) look at lines $y-4=k(x-12)$ and substitute into $(x-5)^2+(y-5)^2=5^2$, i.e. $(x-5)^2+(4+k(x-12)-5)^2=5^2$ which simplifies to a quadratic in $x$ with discriminant $-12k^2-7k+12$ which has solutions $k=-frac{4}{3}$ and $k=frac{3}{4}$. Putting it all together we get $3y+4x-60=0$ and the original line.






      share|cite|improve this answer























        Your Answer





        StackExchange.ifUsing("editor", function () {
        return StackExchange.using("mathjaxEditing", function () {
        StackExchange.MarkdownEditor.creationCallbacks.add(function (editor, postfix) {
        StackExchange.mathjaxEditing.prepareWmdForMathJax(editor, postfix, [["$", "$"], ["\\(","\\)"]]);
        });
        });
        }, "mathjax-editing");

        StackExchange.ready(function() {
        var channelOptions = {
        tags: "".split(" "),
        id: "69"
        };
        initTagRenderer("".split(" "), "".split(" "), channelOptions);

        StackExchange.using("externalEditor", function() {
        // Have to fire editor after snippets, if snippets enabled
        if (StackExchange.settings.snippets.snippetsEnabled) {
        StackExchange.using("snippets", function() {
        createEditor();
        });
        }
        else {
        createEditor();
        }
        });

        function createEditor() {
        StackExchange.prepareEditor({
        heartbeatType: 'answer',
        convertImagesToLinks: true,
        noModals: true,
        showLowRepImageUploadWarning: true,
        reputationToPostImages: 10,
        bindNavPrevention: true,
        postfix: "",
        imageUploader: {
        brandingHtml: "Powered by u003ca class="icon-imgur-white" href="https://imgur.com/"u003eu003c/au003e",
        contentPolicyHtml: "User contributions licensed under u003ca href="https://creativecommons.org/licenses/by-sa/3.0/"u003ecc by-sa 3.0 with attribution requiredu003c/au003e u003ca href="https://stackoverflow.com/legal/content-policy"u003e(content policy)u003c/au003e",
        allowUrls: true
        },
        noCode: true, onDemand: true,
        discardSelector: ".discard-answer"
        ,immediatelyShowMarkdownHelp:true
        });


        }
        });














         

        draft saved


        draft discarded


















        StackExchange.ready(
        function () {
        StackExchange.openid.initPostLogin('.new-post-login', 'https%3a%2f%2fmath.stackexchange.com%2fquestions%2f1608241%2ffind-equation-of-circle-in-the-first-quadrant-touches-x-axis-y-axis-and-stra%23new-answer', 'question_page');
        }
        );

        Post as a guest















        Required, but never shown

























        3 Answers
        3






        active

        oldest

        votes








        3 Answers
        3






        active

        oldest

        votes









        active

        oldest

        votes






        active

        oldest

        votes








        up vote
        0
        down vote













        1) We want to find the $x-$ and $y-$intercepts of the line so that we can use geometry to solve for the radius and center. These are simply $(0, -5)$ and $(frac{20}{3}, 0).$ The hypotenuse of the right triangle formed is $frac{25}{3}$ by Pythagoras. The circle divides each side of the triangle into two parts, and every pair of sides shares a segment. Call the radius $r$ and the other segments $a$ and $b.$ Then we have $r + a = frac{20}{3},$ $r + b = 5,$ and $a + b = frac{25}{3}.$ Solving, we have $a + b + r = 10,$ so $r = 10 - frac{25}{3} = frac{5}{3}.$ This means the center is at $(frac{5}{3},-frac{5}{3}).$ This is actually in Quadrant IV - it is impossible for the circle to be in Quadrant I. The equation of the circle is thus $boxed{(x - frac{5}{3})^{2} + (y + frac{5}{3})^{2} = frac{25}{9}}.$






        share|cite|improve this answer



























          up vote
          0
          down vote













          1) We want to find the $x-$ and $y-$intercepts of the line so that we can use geometry to solve for the radius and center. These are simply $(0, -5)$ and $(frac{20}{3}, 0).$ The hypotenuse of the right triangle formed is $frac{25}{3}$ by Pythagoras. The circle divides each side of the triangle into two parts, and every pair of sides shares a segment. Call the radius $r$ and the other segments $a$ and $b.$ Then we have $r + a = frac{20}{3},$ $r + b = 5,$ and $a + b = frac{25}{3}.$ Solving, we have $a + b + r = 10,$ so $r = 10 - frac{25}{3} = frac{5}{3}.$ This means the center is at $(frac{5}{3},-frac{5}{3}).$ This is actually in Quadrant IV - it is impossible for the circle to be in Quadrant I. The equation of the circle is thus $boxed{(x - frac{5}{3})^{2} + (y + frac{5}{3})^{2} = frac{25}{9}}.$






          share|cite|improve this answer

























            up vote
            0
            down vote










            up vote
            0
            down vote









            1) We want to find the $x-$ and $y-$intercepts of the line so that we can use geometry to solve for the radius and center. These are simply $(0, -5)$ and $(frac{20}{3}, 0).$ The hypotenuse of the right triangle formed is $frac{25}{3}$ by Pythagoras. The circle divides each side of the triangle into two parts, and every pair of sides shares a segment. Call the radius $r$ and the other segments $a$ and $b.$ Then we have $r + a = frac{20}{3},$ $r + b = 5,$ and $a + b = frac{25}{3}.$ Solving, we have $a + b + r = 10,$ so $r = 10 - frac{25}{3} = frac{5}{3}.$ This means the center is at $(frac{5}{3},-frac{5}{3}).$ This is actually in Quadrant IV - it is impossible for the circle to be in Quadrant I. The equation of the circle is thus $boxed{(x - frac{5}{3})^{2} + (y + frac{5}{3})^{2} = frac{25}{9}}.$






            share|cite|improve this answer














            1) We want to find the $x-$ and $y-$intercepts of the line so that we can use geometry to solve for the radius and center. These are simply $(0, -5)$ and $(frac{20}{3}, 0).$ The hypotenuse of the right triangle formed is $frac{25}{3}$ by Pythagoras. The circle divides each side of the triangle into two parts, and every pair of sides shares a segment. Call the radius $r$ and the other segments $a$ and $b.$ Then we have $r + a = frac{20}{3},$ $r + b = 5,$ and $a + b = frac{25}{3}.$ Solving, we have $a + b + r = 10,$ so $r = 10 - frac{25}{3} = frac{5}{3}.$ This means the center is at $(frac{5}{3},-frac{5}{3}).$ This is actually in Quadrant IV - it is impossible for the circle to be in Quadrant I. The equation of the circle is thus $boxed{(x - frac{5}{3})^{2} + (y + frac{5}{3})^{2} = frac{25}{9}}.$







            share|cite|improve this answer














            share|cite|improve this answer



            share|cite|improve this answer








            edited Jan 11 '16 at 23:19

























            answered Jan 11 '16 at 23:14









            K. Jiang

            2,9981412




            2,9981412






















                up vote
                0
                down vote













                Since the circle touches x-axis and y-axis its center must lie on line $x=y$. Its coordinates must be $ (h,h)$. Equation of circle is



                $$ (x-h)^2 +(y-h)^2 = h^2 $$



                It should satisfy line L $$ y = 3 x/4 -5 $$



                Eliminate y:



                $$ x= 4/25 (15+7 h+2 sqrt 2 sqrt{-50-5 h+3 h^2}) $$



                The quantity under radical sign should vanish for a coincident root at tangent point, or ,



                $$ -50-5 h+3 h^2 = 0 $$



                which has roots:



                $$ h = 5 , h = -10/3 $$



                The latter is discarded as the circle must be in first quadrant.



                So the equation of the desired circle is finally:



                $$ (x-5)^2 +(y-5)^2 = 5^2 = 25 $$



                Point of tangential contact $(8,1)$ (not asked for).






                share|cite|improve this answer

























                  up vote
                  0
                  down vote













                  Since the circle touches x-axis and y-axis its center must lie on line $x=y$. Its coordinates must be $ (h,h)$. Equation of circle is



                  $$ (x-h)^2 +(y-h)^2 = h^2 $$



                  It should satisfy line L $$ y = 3 x/4 -5 $$



                  Eliminate y:



                  $$ x= 4/25 (15+7 h+2 sqrt 2 sqrt{-50-5 h+3 h^2}) $$



                  The quantity under radical sign should vanish for a coincident root at tangent point, or ,



                  $$ -50-5 h+3 h^2 = 0 $$



                  which has roots:



                  $$ h = 5 , h = -10/3 $$



                  The latter is discarded as the circle must be in first quadrant.



                  So the equation of the desired circle is finally:



                  $$ (x-5)^2 +(y-5)^2 = 5^2 = 25 $$



                  Point of tangential contact $(8,1)$ (not asked for).






                  share|cite|improve this answer























                    up vote
                    0
                    down vote










                    up vote
                    0
                    down vote









                    Since the circle touches x-axis and y-axis its center must lie on line $x=y$. Its coordinates must be $ (h,h)$. Equation of circle is



                    $$ (x-h)^2 +(y-h)^2 = h^2 $$



                    It should satisfy line L $$ y = 3 x/4 -5 $$



                    Eliminate y:



                    $$ x= 4/25 (15+7 h+2 sqrt 2 sqrt{-50-5 h+3 h^2}) $$



                    The quantity under radical sign should vanish for a coincident root at tangent point, or ,



                    $$ -50-5 h+3 h^2 = 0 $$



                    which has roots:



                    $$ h = 5 , h = -10/3 $$



                    The latter is discarded as the circle must be in first quadrant.



                    So the equation of the desired circle is finally:



                    $$ (x-5)^2 +(y-5)^2 = 5^2 = 25 $$



                    Point of tangential contact $(8,1)$ (not asked for).






                    share|cite|improve this answer












                    Since the circle touches x-axis and y-axis its center must lie on line $x=y$. Its coordinates must be $ (h,h)$. Equation of circle is



                    $$ (x-h)^2 +(y-h)^2 = h^2 $$



                    It should satisfy line L $$ y = 3 x/4 -5 $$



                    Eliminate y:



                    $$ x= 4/25 (15+7 h+2 sqrt 2 sqrt{-50-5 h+3 h^2}) $$



                    The quantity under radical sign should vanish for a coincident root at tangent point, or ,



                    $$ -50-5 h+3 h^2 = 0 $$



                    which has roots:



                    $$ h = 5 , h = -10/3 $$



                    The latter is discarded as the circle must be in first quadrant.



                    So the equation of the desired circle is finally:



                    $$ (x-5)^2 +(y-5)^2 = 5^2 = 25 $$



                    Point of tangential contact $(8,1)$ (not asked for).







                    share|cite|improve this answer












                    share|cite|improve this answer



                    share|cite|improve this answer










                    answered Jan 12 '16 at 16:06









                    Narasimham

                    20.4k52158




                    20.4k52158






















                        up vote
                        0
                        down vote













                        For question 1) we look for circles of the form $(x-a)^2+(y-a)^2=a^2$, since these are tangent to the axes, and the only ones because a tangency to the axis determines the radius line.



                        ($(x-a)^2+(y+a)^2=a^2$ gives the circles in the 2. and 4. quadrants)



                        The problem is to get the third tangency with the given line. First the point needs to be on the circle: $(x-a)^2+((3/4)x-5-a)^2=a^2$ or $(frac{25}{16}x^2+(-frac{7}{2}a-frac{15}{2})x+a^2+10a+25=0$, and it should touch doubly, i.e. the discriminant should be 0. This gives us a 2. order equation in a. In fact it is $(-frac{7}{2}a-frac{15}{2})^2-4(frac{25}{16})(a^2+10a+25)=2 (a - 5) (3 a + 10)=0$, so that $a=5$ or $a=-10/3$ a solution in the third quadrant.



                        Circle bundle varying with a



                        So the solution is $(x-5)^2+(y-5)^2=5^2$ which goes through $(8,1)$ on the line which is a $3,4,5$-triangle from the center $(5,5)$.



                        For question 2) look at lines $y-4=k(x-12)$ and substitute into $(x-5)^2+(y-5)^2=5^2$, i.e. $(x-5)^2+(4+k(x-12)-5)^2=5^2$ which simplifies to a quadratic in $x$ with discriminant $-12k^2-7k+12$ which has solutions $k=-frac{4}{3}$ and $k=frac{3}{4}$. Putting it all together we get $3y+4x-60=0$ and the original line.






                        share|cite|improve this answer



























                          up vote
                          0
                          down vote













                          For question 1) we look for circles of the form $(x-a)^2+(y-a)^2=a^2$, since these are tangent to the axes, and the only ones because a tangency to the axis determines the radius line.



                          ($(x-a)^2+(y+a)^2=a^2$ gives the circles in the 2. and 4. quadrants)



                          The problem is to get the third tangency with the given line. First the point needs to be on the circle: $(x-a)^2+((3/4)x-5-a)^2=a^2$ or $(frac{25}{16}x^2+(-frac{7}{2}a-frac{15}{2})x+a^2+10a+25=0$, and it should touch doubly, i.e. the discriminant should be 0. This gives us a 2. order equation in a. In fact it is $(-frac{7}{2}a-frac{15}{2})^2-4(frac{25}{16})(a^2+10a+25)=2 (a - 5) (3 a + 10)=0$, so that $a=5$ or $a=-10/3$ a solution in the third quadrant.



                          Circle bundle varying with a



                          So the solution is $(x-5)^2+(y-5)^2=5^2$ which goes through $(8,1)$ on the line which is a $3,4,5$-triangle from the center $(5,5)$.



                          For question 2) look at lines $y-4=k(x-12)$ and substitute into $(x-5)^2+(y-5)^2=5^2$, i.e. $(x-5)^2+(4+k(x-12)-5)^2=5^2$ which simplifies to a quadratic in $x$ with discriminant $-12k^2-7k+12$ which has solutions $k=-frac{4}{3}$ and $k=frac{3}{4}$. Putting it all together we get $3y+4x-60=0$ and the original line.






                          share|cite|improve this answer

























                            up vote
                            0
                            down vote










                            up vote
                            0
                            down vote









                            For question 1) we look for circles of the form $(x-a)^2+(y-a)^2=a^2$, since these are tangent to the axes, and the only ones because a tangency to the axis determines the radius line.



                            ($(x-a)^2+(y+a)^2=a^2$ gives the circles in the 2. and 4. quadrants)



                            The problem is to get the third tangency with the given line. First the point needs to be on the circle: $(x-a)^2+((3/4)x-5-a)^2=a^2$ or $(frac{25}{16}x^2+(-frac{7}{2}a-frac{15}{2})x+a^2+10a+25=0$, and it should touch doubly, i.e. the discriminant should be 0. This gives us a 2. order equation in a. In fact it is $(-frac{7}{2}a-frac{15}{2})^2-4(frac{25}{16})(a^2+10a+25)=2 (a - 5) (3 a + 10)=0$, so that $a=5$ or $a=-10/3$ a solution in the third quadrant.



                            Circle bundle varying with a



                            So the solution is $(x-5)^2+(y-5)^2=5^2$ which goes through $(8,1)$ on the line which is a $3,4,5$-triangle from the center $(5,5)$.



                            For question 2) look at lines $y-4=k(x-12)$ and substitute into $(x-5)^2+(y-5)^2=5^2$, i.e. $(x-5)^2+(4+k(x-12)-5)^2=5^2$ which simplifies to a quadratic in $x$ with discriminant $-12k^2-7k+12$ which has solutions $k=-frac{4}{3}$ and $k=frac{3}{4}$. Putting it all together we get $3y+4x-60=0$ and the original line.






                            share|cite|improve this answer














                            For question 1) we look for circles of the form $(x-a)^2+(y-a)^2=a^2$, since these are tangent to the axes, and the only ones because a tangency to the axis determines the radius line.



                            ($(x-a)^2+(y+a)^2=a^2$ gives the circles in the 2. and 4. quadrants)



                            The problem is to get the third tangency with the given line. First the point needs to be on the circle: $(x-a)^2+((3/4)x-5-a)^2=a^2$ or $(frac{25}{16}x^2+(-frac{7}{2}a-frac{15}{2})x+a^2+10a+25=0$, and it should touch doubly, i.e. the discriminant should be 0. This gives us a 2. order equation in a. In fact it is $(-frac{7}{2}a-frac{15}{2})^2-4(frac{25}{16})(a^2+10a+25)=2 (a - 5) (3 a + 10)=0$, so that $a=5$ or $a=-10/3$ a solution in the third quadrant.



                            Circle bundle varying with a



                            So the solution is $(x-5)^2+(y-5)^2=5^2$ which goes through $(8,1)$ on the line which is a $3,4,5$-triangle from the center $(5,5)$.



                            For question 2) look at lines $y-4=k(x-12)$ and substitute into $(x-5)^2+(y-5)^2=5^2$, i.e. $(x-5)^2+(4+k(x-12)-5)^2=5^2$ which simplifies to a quadratic in $x$ with discriminant $-12k^2-7k+12$ which has solutions $k=-frac{4}{3}$ and $k=frac{3}{4}$. Putting it all together we get $3y+4x-60=0$ and the original line.







                            share|cite|improve this answer














                            share|cite|improve this answer



                            share|cite|improve this answer








                            edited Sep 2 '17 at 15:34

























                            answered Jan 11 '16 at 18:50









                            Jan-Magnus Økland

                            1,8461914




                            1,8461914






























                                 

                                draft saved


                                draft discarded



















































                                 


                                draft saved


                                draft discarded














                                StackExchange.ready(
                                function () {
                                StackExchange.openid.initPostLogin('.new-post-login', 'https%3a%2f%2fmath.stackexchange.com%2fquestions%2f1608241%2ffind-equation-of-circle-in-the-first-quadrant-touches-x-axis-y-axis-and-stra%23new-answer', 'question_page');
                                }
                                );

                                Post as a guest















                                Required, but never shown





















































                                Required, but never shown














                                Required, but never shown












                                Required, but never shown







                                Required, but never shown

































                                Required, but never shown














                                Required, but never shown












                                Required, but never shown







                                Required, but never shown







                                Popular posts from this blog

                                Le Mesnil-Réaume

                                Ida-Boy-Ed-Garten

                                web3.py web3.isConnected() returns false always